EMC 2019 Juniors ENG Solutions
EMC 2019 Juniors ENG Solutions
Marin Getaldic
Junior Category
First Solution. Consider an arbitrary marking scheme which follows the given rule.
Let a denote the number of positive integers from the set {1, . . . , 2019} which are marked with a 2, b the number
of those marked with a 1, and c the number of those marked with a 0.
1 point.
We have S = 2a + b.
1 point.
For every positive integer j ∈ {1, . . . , 2017} which is marked with a 2, the number j + 2 is marked with a 0. This implies
that the number of positive integers less than 2017 marked with 2 is less than or equal to c.
1 point.
Hence, this implies a 6 c + 2. We then have
S = 2a + b 6 a + b + c + 2 = 2019 + 2 = 2021.
3 points.
Consider the following marking scheme:
Here the i-th digit in the sequence denotes the mark of positive integer i. For this marking, S = 2021, and therefore
2021 is the maximum possible value of S.
4 points.
1
Second Solution. The marking scheme for which S = 2021 is the same as in the first solution.
4 points.
Let Sn denote the sum of marks of first n positive integers, and let ak denote the mark of k. Without loss of generality
we may assume aj = 0 for all integers j 6 0. We’ll prove the following claim by strong mathematical induction:
1 point.
The base cases for n ∈ {1, 2} trivially hold. Suppose the claim is true for all k 6 n for some n > 2.
Suppose there exists a marking scheme for which Sn+1 > n + 4. Then if an+1 < 2, we have Sn > n + 3, which is a
contradiction. Hence, an+1 = 2.
1 point.
This implies that an ∈ {0, 2}. If an = 0, then Sn−1 > n + 2, which is a contradiction. So, an = 2.
1 point.
Now an−1 = 0 because both an and an+1 are nonzero. We now have Sn−2 > n, and by the induction hypothesis, it must
hold that Sn−2 = n and an−2 = 2. However, this is in contradiction with an being nonzero. Hence, Sn+1 6 n + 3.
1 point.
Suppose Sn+1 = n + 3 and an+1 6= 2. If an+1 = 0, then Sn > n + 3, which is a contradiction. Thus, an+1 = 1.
1 point.
Then Sn = n + 2, which implies an = 2. Then we must have an−1 = 0, and then Sn−2 = n, which implies an−2 = 2,
but an is nonzero, which is a contradiction. Therefore, the claim is true for n + 1, which implies it is true for all positive
integers. In particular, S2019 6 2021, which combined with the construction implies that the maximum value of S is
2021.
1 point.
Notes on marking:
• If a student forgets to write additional zeros beyond the first 2019 digits in his construction, but the construction
is otherwise valid, he should be awarded all 4 points for this part.
• There are many different optimal marking schemes. For example, 2200|210|210| . . . |210|22|000 . . ., where the block
|210| repeats 671 times.
• In the Second Solution, if the student writes only the first part of the induction hypothesis without the assumption
that an = 2 in the case of equality: he should be awarded 0 points, unless he reaches additional conclusions which
lead to the solution.
• In the Second Solution, if the student doesn’t comment on the base case/cases at all, he should be deducted 1
point.
• If the student proves any nontrivial lemma useful for any of the solutions, but the lemma itself isn’t worth any
points and the student wouldn’t otherwise get any of the 6 points given for proving the bound, he should get 1
point for this part.
2
√
Problem 2. Let (xn )n∈N be a sequence defined recursively such that x1 = 2 and
1
xn+1 = xn + for n ∈ N.
xn
Prove that the following inequality holds:
(Ivan Novak)
1 point.
By squaring the given assertion, we get the equality 2 + 1
x2
= x2n+1 − x2n . This implies that the left hand side equals
n
1 1 1 1
+ 2 + ... + 2 + 2 .
x22 − x21 x3 − x22 x2019 − x22018 x2020 − x22019
1 point.
Using the inequality between arithmetic and harmonic mean, we find that the left hand side is greater than or equal to
20192
.
(x22 − x21 ) + (x23 − x22 ) + . . . + (x22020 − x22019 )
4 points.
We now notice that the denominator is a telescoping sum and it equals x22020 − x21 , which implies the right hand side
equals
20192 20192
= ,
x22020 − x21 x22020 − 2
which is exactly equal to the right hand side.
1 point.
The equality cannot hold because x22 − x21 6= x23 − x22 .
1 point.
3
Second Solution. As in the first solution, we obtain that the left hand side equals
1 1 1 1
1 + 1 + ... + + .
2 + x2 2 + x2 2 + x21 2+ x2
1
1 2 2018 2019
2 points.
Using the inequality between arithmetic and harmonic mean, we get that the left hand side is greater than or equal to
20192
1 .
2 · 2019 + x2
+ x12 + . . . + x2
1
1 2 2019
4 points.
We now prove by mathematical induction that
1 1 1
2·n+ + 2 + ... + 2 = x2n
x21 x2 xn−1
where we used the induction hypothesis for the last equality. This proves the claim.
2 points.
In particular, for n = 2019, we have that
20192 20192
= ,
2 · 2019 + x12 + x12 + . . . + x2
1
x22019 + x21
1 2 2019 2019
1 1
The equality cannot hold because x2
+ 2 6= x2
+ 2.
1 2
1 point.
4
Third Solution. We prove by mathematical induction that for every n > 2 the following inequality holds:
2 4.5 17 4 72 17
For n = 2, the left hand side equals 5
+ 10
= 20
, and the right hand side equals 9+2 = 85
< 20
, which proves the base
2 9
case.
Suppose the claim was true for some n ∈ N. Then by the induction hypothesis, we know that
1 point.
We now prove that 2xn+1 xn+2 − 1 = 2x2n+1 + 1 as in the first solution.
1 point.
We then have
n2 x2n+1 n2 x2n+1 n2 1
1 + = 2 1 + = 2 +
1
.
x2n + x2 2x x
n+1 n+2 − 1 xn + x2 2x2
n+1 + 1 xn + x12 2+ 2
n n n
xn+1
1 point.
By the inequality of arithmetic and harmonic mean, this is greater than or equal to
(n + 1)2
.
x2n + x12 + 2 + x2
1
n n+1
5 points.
1
Notice that squaring the assertion xn+1 = xn + xn
, we obtain
1
x2n + + 2 = x2n+1 .
x2n
1 point.
This implies that
(n + 1)2 (n + 1)2
= ,
x2n + x12 + 2 + x2
1
x2n+1+ x21
n n+1 n+1
which is exactly equal to the right hand side. Therefore, the claim is proven by the principle of mathematical induction.
In particular, the claim is true for n = 2019, which proves the inequality.
1 point.
Notes on marking:
• Points from separate solutions can not be added. The student should be awarded the maximum of the points scored
in the 3 presented solutions, or an appropriate number of points on an alternative solution.
• The third solution gives 5 points for the use of AM-HM inequality as opposed to 4 points in the first solution
because in the third solution it is not necessary to comment the equality case. However, if a student has n = 1 as
a basis of induction and doesn’t comment the equality case, he should be deducted 1 point out of possible 5.
• The point for proving that the equality cannot be achieved is only awarded if the student has proved the non-strict
version of inequality.
5
Problem 3. Let ABC be a triangle with circumcircle ω. Let lB and lC be two lines through the points B and
C, respectively, such that lB k lC . The second intersections of lB and lC with ω are D and E, respectively.
Assume that D and E are on the same side of BC as A. Let DA intersect lC at F and let EA intersect lB
at G. If O, O1 and O2 are circumcenters of the triangles ABC, ADG and AEF , respectively, and P is the
circumcenter of the triangle OO1 O2 , prove that lB k OP k lC .
(Stefan Lozanovski)
Sketch.
O2
E
A
S
G P
O1
B C
lB lC
Lemma. Triangles AGD and AEF are similar to the triangle ABC.
Proof. As DBCAE is a cylic pentagon we have
∠GDA = ∠BCA = γ.
1 point.
Now from lB k lC we get that
∠DBA = ∠DBC − β = 180◦ − ∠BCE − β = α + γ − ∠BCE = α − ∠ACE
1 point.
so from the cyclicity
∠BCD = ∠BAD = 180◦ − ∠DBA − ∠ADB = 180◦ − (α − ∠ACE) − (180◦ − γ) = γ − α + ∠ACE
1 point.
Hence
∠DAG = ∠DCE = ∠BCA − ∠BCD + ∠ACE = α
1 point.
Therefore AGD is similar to the triangle ABC, and similarly for AEF .
6
Now as G, A and E are collinear and F , A and D are collinear, using Lemma we get that O, O1 and O2 are collinear.
1 point.
As O1 is the circumcenter of the triangle ADG and O1 D is the bisector of the chord AD we get that
1
∠AO1 O = ∠AO1 D = ∠AGD = β
2
and similarly ∠AO1 O = γ, so the triangle OO1 O2 is similar to the triangle ABC.
2 points.
Now as P is the circumcenter of the triangle OO1 O2 from the previous similarity we get that
∠O1 OP = ∠BAO
1 point.
Hence
∠DOP = ∠DOO1 + ∠O1 OP = ∠DBA + ∠BAO = ∠DBA + ∠ABO = ∠DBO = ∠ODB
so lB k OP k lC .
2 points.
Notes on marking:
• If a student has a partial solution with analytic methods, only points for proving facts that can be expressed in
geometric ways and lead to a compete solution can be awarded.
7
Problem 4. Let u be a positive rational number and m be a positive integer. Define a sequence q1 , q2 , q3 , . . .
such that q1 = u and for n > 2:
a a + mb
if qn−1 = for some relatively prime positive integers a and b, then qn = .
b b+1
Determine all positive integers m such that the sequence q1 , q2 , q3 , . . . is eventually periodic for any positive
rational number u.
Remark: A sequence x1 , x2 , x3 , . . . is eventually periodic if there are positive integers c and t such that xn = xn+t
for all n > c.
(Petar Nizić-Nikolac)
Solution. We will prove that the sequence is eventually periodic if and only if m is odd.
Let a1 , a2 , a3 , . . . and b1 , b2 , b3 , . . . be sequences of numerators and denumerators of q1 , q2 , q3 , . . . respectively when written
in the irreducible form, i.e. for n ∈ N:
an
qn = gcd(an , bn ) = 1
bn
Say that there was reduction in the nth step if gcd(an + mbn , bn + 1) > 1.
Case 1. m is even
Set u = 11 . Assume for the sake of contradiction that q1 , q2 , q3 , . . . is eventually periodic. Then (bn )n∈N is bounded so
there is r > 1 (pick the smallest one) such that there was reduction in the rth step. Easy to see that
r
1 m+1 3m + 1 6m + 1 10m + 1 m+1
q1 = , q 2 = , q3 = , q4 = , q5 = , . . . , qr = 2
1 2 3 4 5 r
2 points.
Now as m is even we have
! ! ! !
r r+1 m
gcd (ar + mbr , br + 1) = gcd m + 1 + mr, r + 1 = gcd m + 1, r + 1 = gcd (r + 1)r + 1, r + 1 = 1
2 2 2
so this is a contradiction, and hence it is not eventually periodic for any positive rational number u.
1 point.
Case 2. m is odd
Assume that there is r ∈ N such that there was no reduction in the steps r, r + 1, r + 2 and r + 3. Then for i ∈ {1, 2}:
(ar+i+2 , br+i+2 ) ≡ (ar+i + mbr+i + mbr+i+1 , br+i + 1 + 1) ≡ (ar+i + 2mbr+i + m, br+i + 2) ≡ (ar+i + 1, br+i ) (mod 2)
so at least one of the following pairs: (ar+1 , br+1 ), (ar+2 , br+2 ), (ar+3 , br+3 ), (ar+4 , br+4 ) has both even entries which is
impossible (as they are coprime). Hence there was at least one reduction in the steps r, r + 1, r + 2 and r + 3.
2 points.
Therefore for all n > 1:
1
max{bn+1 , bn+2 , bn+3 , bn+4 } 6 min{bn+1 , bn+2 , bn+3 , bn+4 } + 3 6 max{bn , bn+1 , bn+2 , bn+3 } + 3
2
so there exists C > 1 such that bn 6 6 for all n > C.
2 points.
Similarly for all n > C:
1
max{an+1 , an+2 , an+3 , an+4 } 6 min{an+1 , an+2 , an+3 , an+4 } + 3 · 6m 6 max{an , an+1 , an+2 , an+3 } + 18m
2
so there exists D > C such that an 6 36m for all n > D.
2 points.
We conclude that for all n > D there are finitely many pairs (6 · 36m = 216m) that (an , bn ) attains so it becomes
eventually periodic for any positive rational number u.
1 point.
Notes on marking:
• Case 1 and Case 2 are always worth 3 points and 7 points respectively.